• 具体数学第二版第四章习题(3)


    31 $(b)mod(d)=1 ightarrow (b^{m})mod(d)=((kd+1)^{m})mod(d)=1$

    所以$((a_{m}a_{m-1}...a_{1}a_{0})_{b}=sum_{k=0}^{m}a_{k}b^{k})mod(d)=sum_{k=0}^{m}a_{k}$ 也就是说,只要$(b)mod(d)=1$,那么一个$b$进制的数字能够被$d$整除当且仅当各位数字之和能够被$d$整除

    32 $left { (kn)mod(m)|kperp m ,0le k <m ight }=left { (k)mod(m)|kperp m ,0le k <m ight }$,所以将两边的$varphi (m)$个数字乘起来,两边除以$prod _{kperp m ,0le k <m}k$即可

    33 $h(1)=1$,假设$nperp m$,那么$h(mn)=sum_{dperp mn}f(d)g(frac{mn}{d})=sum_{xperp m,yperp n}f(xy)g(frac{m}{x}frac{n}{y})=sum_{xperp m}sum_{yperp n}f(x)g(frac{m}{x})f(y)g(frac{n}{y})=h(n)h(m)$

    34 在公式4.56中,如果$d$不是整数,那么$f(d)=0$,所以$g(m)=sum_{d|m}f(d)=sum_{d|m}f(frac{m}{d})=sum_{dgeq 1}f(frac{m}{d})$

    35 下面使用的符号与公式4.5相关的符号相同。

    $m^{'}=overline{m}-left lfloor frac{n}{m} ight flooroverline{r},n^{'}=overline{r} ightarrow I(m,n)=m^{'}=I(m,overline{r})-left lfloor frac{n}{m} ight floor I(overline{r},m)=I(m,(n)mod(m))-left lfloor frac{n}{m} ight floor I((n)mod(m),m),I(n,m)=n^{'}=(n)mod(m)$

    $I(0,n)=0,I(m,0)=1$

    36 首先证明2不可以。

    假设2可以分解为两个非单位数乘积,即$2=(a+bsqrt{10})(c+dsqrt{10})=(ac+10bd)+(ad+bc)sqrt{10} ightarrow ac+10bd=2,ad+bc=0 ightarrow (a-bsqrt{10})(c-dsqrt{10})=(ac+10bd)-(ad+bc)sqrt{10}=2 ightarrow (a^{2}-10b^{2})(c^{2}-10d^{2})=4$

    由于$|a^{2}-10b^{2}| eq 1,|c^{2}-10d^{2}| eq 1$,所以要么它们都为2或者都为-2. 由于任何整数的平方模10为0,1,4,5,6,9,所以不会是2.所以$(a^{2}-10b^{2})mod(10)=(a^{2})mod(10) eq 2$。所以假设错误。

    3和$4pm sqrt{10}$的证明类似

    37 令$a_{n}=2^{-n}ln(e_{n}-frac{1}{2})=2^{-n}ln((e_{n-1}-frac{1}{2})^{2}+frac{1}{4})>2^{-n}ln((e_{n-1}-frac{1}{2})^{2})=2^{-(n-1)}ln(e_{n-1}-frac{1}{2})=a_{n-1}$

    令$b_{n}=2^{-n}ln(e_{n}+frac{1}{2})=2^{-n}ln(e_{n-1}^{2}-e_{n-1}+frac{3}{2})<2^{-n}ln(e_{n-1}^{2}+e_{n-1}+frac{1}{4})=2^{-n}ln((e_{n-1}+frac{1}{2})^{2})=b_{n-1}$

    所以$a_{n-1}<a_{n}<b_{n}<b_{n-1}$

    另外$e_{n}=left lfloor E^{2^{n}}+frac{1}{2} ight floorLeftrightarrow e_{n}leq E^{2^{n}}+frac{1}{2}<e_{n}+1Leftrightarrow e_{n}-frac{1}{2}leq E^{2^{n}}<e_{n}+frac{1}{2}Leftrightarrow 2^{-n}ln(e_{n}-frac{1}{2})leq lnE<2^{-n}ln(e_{n}+frac{1}{2})Leftrightarrow a_{n}leq lnE<b_{n}$

    所以$E=lim_{n ightarrow oo}e^{a_{n}}$

    38 令$r=(n)mod(m)$,那么$a^{n}-b^{n}=(a^{m}-b^{m})(a^{n-m}+a^{n-2m}b^{m}+...+a^{r}b^{n-m-r})+b^{mleft lfloor frac{n}{m} ight floor}(a^{r}-b^{r})$

    所以$Gcd(a^{n}-b^{n},a^{m}-b^{m})=Gcd((a^{n}-b^{n})mod(a^{m}-b^{m}),a^{m}-b^{m})=Gcd(b^{mleft lfloor frac{n}{m} ight floor}(a^{r}-b^{r}),a^{m}-b^{m})$

    因为$aperp b ightarrow b^{m}perp (a^{m}-b^{m}) ightarrow b^{mleft lfloor frac{n}{m} ight floor}perp (a^{m}-b^{m})$

    所以$Gcd(b^{mleft lfloor frac{n}{m} ight floor}(a^{r}-b^{r}),a^{m}-b^{m})=Gcd(a^{r}-b^{r},a^{m}-b^{m})$

    一直这样下去可以得到$Gcd(a^{n}-b^{n},a^{m}-b^{m})=a^{Gcd(n,m)}-b^{Gcd(n,m)}$

    39 假设相等,设$m$的序列为$S_{m}=left { m,a_{1},a_{2},...,a_{t},S(m) ight }$,$m^{'}$的序列为$S_{m^{'}}=left { m^{'},b_{1},b_{2},...,b_{u},S(m) ight }$,令$left { m,a_{1},a_{2},...,a_{t},S(m) ight }cap left { m^{'},b_{1},b_{2},...,b_{u},S(m) ight }=left { c_{1},c_{2},..,c_{k},S_{m} ight }=U$

    所以$frac{prod_{xin S_{m}}xprod_{yin S_{m^{'}}}y}{prod_{xin U}x^{2}}$也是完全平方数,所以这时候$S(m)$不是最小的

    40 这里的$p$是一个素数。

    令$f(n)=prod_{1leq k leq n,(k)mod(p) e 0}k=frac{n!}{p^{left lfloor frac{n}{p} ight floor}left lfloor frac{n}{p} ight floor!}$

    那么$frac{n!}{p^{xi _{p}(n!)}}=f(n)f(left lfloor frac{n}{p} ight floor)f(left lfloor frac{n}{p^{2}} ight floor)...$

    而$f(n)equiv a_{0}!((p-1)!)^{left lfloor frac{n}{p} ight floor}equiv a_{0}!(-1)^{left lfloor frac{n}{p} ight floor}(mod(p))$

    $f(left lfloor frac{n}{p} ight floor)equiv a_{1}!(-1)^{left lfloor frac{n}{p^{2}} ight floor}(mod(p))$

    $f(left lfloor frac{n}{p^{2}} ight floor)equiv a_{2}!(-1)^{left lfloor frac{n}{p^{3}} ight floor}(mod(p))$

    乘起来可以得到$frac{n!}{p^{xi _{p}(n!)}}equiv (-1)^{xi _{p}(n!)}a_{0}!a_{1}!..a_{m}!(mod(p))$

    41 (1)假设$n^{2}equiv -1(mod(p)) ightarrow (n^{2})^{frac{p-1}{2}}=n^{p-1}equiv -1(mod(p))$ ,这是矛盾的

    (2)$n=(frac{p-1}{2})!$

    比如$n=13$,那么$(1)mod(13)=(-12)mod(13),(2)mod(13)=(-11)mod(13),..,(6)mod(13)=(-7)mod(13),$

    所以$nequiv left ( (-1)^{frac{p-1}{2}}prod_{1leq k leq frac{p-1}{2}}(p-k)=frac{(p-1)!}{n} ight )(mod(p)) ightarrow n^{2}equiv left ((p-1)!=1  ight )(mod(p))$

    42 如果$Gcd(mn^{'}+nm^{'},nn^{'})=a>1$,假设$a$可以整除$n$,那么$a$不能整除$n^{'}$.所以$a$可以整除$nm^{'}$,所以$a$也可以整除$mn^{'}$,而$a$不能整除$m$也不能整除$n^{'}$,所以这是不成立的

    43 函数$ ho (n)$在这里的题目23中。

    很明显,奇数次乘以的矩阵一定是$L^{-1}R$。偶数次$n=2k$乘以的矩阵是$R^{- ho (n)}L^{-1}RL^{ ho(n)}$.这个矩阵乘以前以及以后的样子是$...Lunderbrace{RR...R}_{ ho(n)} ightarrow ...Runderbrace{LL...L}_{ ho(n)}$

    44 数字0.3155和0.3165的分数是$frac{631}{2000},frac{633}{2000}$。在Stern-Brocot中在这个区间中最简单的分数是$frac{6}{19}$

    45 $x^{2}equiv x(mod(10^{n}))Leftrightarrow x(x-1)equiv (mod(10^{n}))Leftrightarrow x(x-1)equiv (mod(2^{n})),x(x-1)equiv (mod(5^{n}))Leftrightarrow (x)mod(2^{n})=0,1,(x)mod(5^{n})=0,1$

    所以满足条件的$x$最多有四个,其中两个是$0,1$,另外两个的形式为$t,10^{n}+1-t$

  • 相关阅读:
    英语语法
    elk笔记2.0
    zabbix4.0-centos6 报错坑,是因为有3.0的包
    jenkins 用户授权
    python随记
    jenkins
    zabbix3.0、4.0 agent安装配置
    常用
    document.createElement()方法
    581. Shortest Unsorted Continuous Subarray(LeetCode)
  • 原文地址:https://www.cnblogs.com/jianglangcaijin/p/9838450.html
Copyright © 2020-2023  润新知